How to find this particular probability?

  • Thread starter Thread starter dexterdev
  • Start date Start date
  • Tags Tags
    Probability
Click For Summary
To find the probability P(X1 + X2 < X3) for three independent uniformly distributed random variables X1, X2, and X3 in the range [0,1], the integral limits are derived from the conditions of the problem. The outer integral for X3 ranges from 0 to 1, while the limits for X2 depend on X3, ranging from 0 to X3. The innermost integral for X1 is limited from 0 to X3 - X2, reflecting the condition X1 + X2 < X3. The theoretical solution results in a probability of 1/6, or approximately 0.1667, while a MATLAB simulation yielded a slightly different result of around 0.169. Understanding the limits of integration is crucial for solving this probability problem correctly.
dexterdev
Messages
194
Reaction score
1

Homework Statement



Suppose there are three statistically i.i.d continuous random variables X1, X2, X3 each are uniformly distributed in the range [0,1]. How to find the probability P(X1+X2<X3)?

Homework Equations


The below given equations are the steps to the solution. But I can't figure out how the limits of integral comes this way.

\int_0^1 \int_0^{x_3}\int_0^{x_3-x_2} \,dx_1\,dx_2\,dx_3 =\int_0^1 \int_0^{x_3} (x_3-x_2) dx_2\,dx_3 = \int_0^1 x_3^2 - \frac{x_3^2}{2}\,dx_3 = \frac16 = 0.1\overline 6


The Attempt at a Solution



I tried this using a software called MATLAB by generating three pseudo random variables (1000 samples) and finding X1+X2−X3 and plotting its CDF through a MATLAB tool called dfittool. I got the answer around 0.169. But how do I do this theoretically? Especially how to figure out the limits in those integrals?
 
Physics news on Phys.org
Your questions make no sense: you have already obtained the answer theoretically, and you have already written the limits of integration.
 
@Ray Vickson : Yes I have got those limits from someone else, but never told how they come?
 
They come from two different concerns, that x1+x2<x3 and that each xi must be between 0 and 1. Those integration limits represent the intersection of those concerns.
 
  • Like
Likes 1 person
You are told that the three variables all lie in [0, 1]. The limits on the outer integral, with respect to x_3 must be constants so must be 0 and 1. The next inner integral can have limits depending on x_3. Since we have x_1+ x_2&lt; x_3 and x_1 can be 0, x_2 can go from 0 to x_3. Finally, x_1+ x_2&lt; x_3 means that x_1&lt; x_3- x_2 so the inmost integral has limits of 0 to x_3- x_2.
 
  • Like
Likes 1 person
Question: A clock's minute hand has length 4 and its hour hand has length 3. What is the distance between the tips at the moment when it is increasing most rapidly?(Putnam Exam Question) Answer: Making assumption that both the hands moves at constant angular velocities, the answer is ## \sqrt{7} .## But don't you think this assumption is somewhat doubtful and wrong?

Similar threads

  • · Replies 9 ·
Replies
9
Views
1K
  • · Replies 14 ·
Replies
14
Views
3K
  • · Replies 3 ·
Replies
3
Views
1K
  • · Replies 6 ·
Replies
6
Views
2K
  • · Replies 12 ·
Replies
12
Views
4K
  • · Replies 5 ·
Replies
5
Views
3K
  • · Replies 8 ·
Replies
8
Views
2K
  • · Replies 11 ·
Replies
11
Views
2K
  • · Replies 6 ·
Replies
6
Views
1K
Replies
3
Views
2K